Use the concavity of $log (1-x)+x$ to show that $log (1-x)leq -x$












1












$begingroup$


I've been reading this post and its accepted answer here. The OP (accepted answer) made a comment that $log(1-x)leq -x$ but I've been having issues proving it.



FULL PROOF (EDIT)



With credits to Kavi Rama Murthy, I provide a full proof.



Let $f: ]0,1[longrightarrow Bbb{R},;x mapsto log (1-x)+x$. Then,



begin{align}f''(x) = -frac{1}{(x-1)^2}leq 0,;forall ;xin ;]0,1[.end{align}
Hence, $f$ is monotone non-increasing and $f(x)leq f(0),forall ;xin ;]0,1[,$ that is
begin{align}log(1-x)leq - x end{align}










share|cite|improve this question











$endgroup$








  • 3




    $begingroup$
    $log(1-x)$ is not defined for $xgeq 1$
    $endgroup$
    – Kavi Rama Murthy
    Jan 3 at 8:53










  • $begingroup$
    To show that $f(x)leq f(0),forall ;xin ]0,1[,$ i.e. $log(1-x)+xleq 0$
    $endgroup$
    – Omojola Micheal
    Jan 3 at 9:13


















1












$begingroup$


I've been reading this post and its accepted answer here. The OP (accepted answer) made a comment that $log(1-x)leq -x$ but I've been having issues proving it.



FULL PROOF (EDIT)



With credits to Kavi Rama Murthy, I provide a full proof.



Let $f: ]0,1[longrightarrow Bbb{R},;x mapsto log (1-x)+x$. Then,



begin{align}f''(x) = -frac{1}{(x-1)^2}leq 0,;forall ;xin ;]0,1[.end{align}
Hence, $f$ is monotone non-increasing and $f(x)leq f(0),forall ;xin ;]0,1[,$ that is
begin{align}log(1-x)leq - x end{align}










share|cite|improve this question











$endgroup$








  • 3




    $begingroup$
    $log(1-x)$ is not defined for $xgeq 1$
    $endgroup$
    – Kavi Rama Murthy
    Jan 3 at 8:53










  • $begingroup$
    To show that $f(x)leq f(0),forall ;xin ]0,1[,$ i.e. $log(1-x)+xleq 0$
    $endgroup$
    – Omojola Micheal
    Jan 3 at 9:13
















1












1








1


1



$begingroup$


I've been reading this post and its accepted answer here. The OP (accepted answer) made a comment that $log(1-x)leq -x$ but I've been having issues proving it.



FULL PROOF (EDIT)



With credits to Kavi Rama Murthy, I provide a full proof.



Let $f: ]0,1[longrightarrow Bbb{R},;x mapsto log (1-x)+x$. Then,



begin{align}f''(x) = -frac{1}{(x-1)^2}leq 0,;forall ;xin ;]0,1[.end{align}
Hence, $f$ is monotone non-increasing and $f(x)leq f(0),forall ;xin ;]0,1[,$ that is
begin{align}log(1-x)leq - x end{align}










share|cite|improve this question











$endgroup$




I've been reading this post and its accepted answer here. The OP (accepted answer) made a comment that $log(1-x)leq -x$ but I've been having issues proving it.



FULL PROOF (EDIT)



With credits to Kavi Rama Murthy, I provide a full proof.



Let $f: ]0,1[longrightarrow Bbb{R},;x mapsto log (1-x)+x$. Then,



begin{align}f''(x) = -frac{1}{(x-1)^2}leq 0,;forall ;xin ;]0,1[.end{align}
Hence, $f$ is monotone non-increasing and $f(x)leq f(0),forall ;xin ;]0,1[,$ that is
begin{align}log(1-x)leq - x end{align}







real-analysis analysis logarithms






share|cite|improve this question















share|cite|improve this question













share|cite|improve this question




share|cite|improve this question








edited Jan 3 at 9:25







Omojola Micheal

















asked Jan 3 at 8:51









Omojola MichealOmojola Micheal

1,809324




1,809324








  • 3




    $begingroup$
    $log(1-x)$ is not defined for $xgeq 1$
    $endgroup$
    – Kavi Rama Murthy
    Jan 3 at 8:53










  • $begingroup$
    To show that $f(x)leq f(0),forall ;xin ]0,1[,$ i.e. $log(1-x)+xleq 0$
    $endgroup$
    – Omojola Micheal
    Jan 3 at 9:13
















  • 3




    $begingroup$
    $log(1-x)$ is not defined for $xgeq 1$
    $endgroup$
    – Kavi Rama Murthy
    Jan 3 at 8:53










  • $begingroup$
    To show that $f(x)leq f(0),forall ;xin ]0,1[,$ i.e. $log(1-x)+xleq 0$
    $endgroup$
    – Omojola Micheal
    Jan 3 at 9:13










3




3




$begingroup$
$log(1-x)$ is not defined for $xgeq 1$
$endgroup$
– Kavi Rama Murthy
Jan 3 at 8:53




$begingroup$
$log(1-x)$ is not defined for $xgeq 1$
$endgroup$
– Kavi Rama Murthy
Jan 3 at 8:53












$begingroup$
To show that $f(x)leq f(0),forall ;xin ]0,1[,$ i.e. $log(1-x)+xleq 0$
$endgroup$
– Omojola Micheal
Jan 3 at 9:13






$begingroup$
To show that $f(x)leq f(0),forall ;xin ]0,1[,$ i.e. $log(1-x)+xleq 0$
$endgroup$
– Omojola Micheal
Jan 3 at 9:13












2 Answers
2






active

oldest

votes


















3












$begingroup$

The function is concave on $(0,1)$ because $f''(x)=-frac 1 {(x-1)^{2}} leq 0$. It is not defined for $x geq 1$.






share|cite|improve this answer











$endgroup$













  • $begingroup$
    (+1) But sorry for the stress!
    $endgroup$
    – Omojola Micheal
    Jan 3 at 9:07





















1












$begingroup$

Another way, is to consider the Maclaurin series expansion



begin{align} log(1-x)=-sum^{infty}_{n=1}dfrac{x^n}{n}=-x-dfrac{x^2}{2}-dfrac{x^3}{3}cdots leq -x,;text{for fixed};xin;]0,1[end{align}






share|cite|improve this answer









$endgroup$













    Your Answer





    StackExchange.ifUsing("editor", function () {
    return StackExchange.using("mathjaxEditing", function () {
    StackExchange.MarkdownEditor.creationCallbacks.add(function (editor, postfix) {
    StackExchange.mathjaxEditing.prepareWmdForMathJax(editor, postfix, [["$", "$"], ["\\(","\\)"]]);
    });
    });
    }, "mathjax-editing");

    StackExchange.ready(function() {
    var channelOptions = {
    tags: "".split(" "),
    id: "69"
    };
    initTagRenderer("".split(" "), "".split(" "), channelOptions);

    StackExchange.using("externalEditor", function() {
    // Have to fire editor after snippets, if snippets enabled
    if (StackExchange.settings.snippets.snippetsEnabled) {
    StackExchange.using("snippets", function() {
    createEditor();
    });
    }
    else {
    createEditor();
    }
    });

    function createEditor() {
    StackExchange.prepareEditor({
    heartbeatType: 'answer',
    autoActivateHeartbeat: false,
    convertImagesToLinks: true,
    noModals: true,
    showLowRepImageUploadWarning: true,
    reputationToPostImages: 10,
    bindNavPrevention: true,
    postfix: "",
    imageUploader: {
    brandingHtml: "Powered by u003ca class="icon-imgur-white" href="https://imgur.com/"u003eu003c/au003e",
    contentPolicyHtml: "User contributions licensed under u003ca href="https://creativecommons.org/licenses/by-sa/3.0/"u003ecc by-sa 3.0 with attribution requiredu003c/au003e u003ca href="https://stackoverflow.com/legal/content-policy"u003e(content policy)u003c/au003e",
    allowUrls: true
    },
    noCode: true, onDemand: true,
    discardSelector: ".discard-answer"
    ,immediatelyShowMarkdownHelp:true
    });


    }
    });














    draft saved

    draft discarded


















    StackExchange.ready(
    function () {
    StackExchange.openid.initPostLogin('.new-post-login', 'https%3a%2f%2fmath.stackexchange.com%2fquestions%2f3060375%2fuse-the-concavity-of-log-1-xx-to-show-that-log-1-x-leq-x%23new-answer', 'question_page');
    }
    );

    Post as a guest















    Required, but never shown

























    2 Answers
    2






    active

    oldest

    votes








    2 Answers
    2






    active

    oldest

    votes









    active

    oldest

    votes






    active

    oldest

    votes









    3












    $begingroup$

    The function is concave on $(0,1)$ because $f''(x)=-frac 1 {(x-1)^{2}} leq 0$. It is not defined for $x geq 1$.






    share|cite|improve this answer











    $endgroup$













    • $begingroup$
      (+1) But sorry for the stress!
      $endgroup$
      – Omojola Micheal
      Jan 3 at 9:07


















    3












    $begingroup$

    The function is concave on $(0,1)$ because $f''(x)=-frac 1 {(x-1)^{2}} leq 0$. It is not defined for $x geq 1$.






    share|cite|improve this answer











    $endgroup$













    • $begingroup$
      (+1) But sorry for the stress!
      $endgroup$
      – Omojola Micheal
      Jan 3 at 9:07
















    3












    3








    3





    $begingroup$

    The function is concave on $(0,1)$ because $f''(x)=-frac 1 {(x-1)^{2}} leq 0$. It is not defined for $x geq 1$.






    share|cite|improve this answer











    $endgroup$



    The function is concave on $(0,1)$ because $f''(x)=-frac 1 {(x-1)^{2}} leq 0$. It is not defined for $x geq 1$.







    share|cite|improve this answer














    share|cite|improve this answer



    share|cite|improve this answer








    edited Jan 3 at 9:08

























    answered Jan 3 at 8:56









    Kavi Rama MurthyKavi Rama Murthy

    53.8k32055




    53.8k32055












    • $begingroup$
      (+1) But sorry for the stress!
      $endgroup$
      – Omojola Micheal
      Jan 3 at 9:07




















    • $begingroup$
      (+1) But sorry for the stress!
      $endgroup$
      – Omojola Micheal
      Jan 3 at 9:07


















    $begingroup$
    (+1) But sorry for the stress!
    $endgroup$
    – Omojola Micheal
    Jan 3 at 9:07






    $begingroup$
    (+1) But sorry for the stress!
    $endgroup$
    – Omojola Micheal
    Jan 3 at 9:07













    1












    $begingroup$

    Another way, is to consider the Maclaurin series expansion



    begin{align} log(1-x)=-sum^{infty}_{n=1}dfrac{x^n}{n}=-x-dfrac{x^2}{2}-dfrac{x^3}{3}cdots leq -x,;text{for fixed};xin;]0,1[end{align}






    share|cite|improve this answer









    $endgroup$


















      1












      $begingroup$

      Another way, is to consider the Maclaurin series expansion



      begin{align} log(1-x)=-sum^{infty}_{n=1}dfrac{x^n}{n}=-x-dfrac{x^2}{2}-dfrac{x^3}{3}cdots leq -x,;text{for fixed};xin;]0,1[end{align}






      share|cite|improve this answer









      $endgroup$
















        1












        1








        1





        $begingroup$

        Another way, is to consider the Maclaurin series expansion



        begin{align} log(1-x)=-sum^{infty}_{n=1}dfrac{x^n}{n}=-x-dfrac{x^2}{2}-dfrac{x^3}{3}cdots leq -x,;text{for fixed};xin;]0,1[end{align}






        share|cite|improve this answer









        $endgroup$



        Another way, is to consider the Maclaurin series expansion



        begin{align} log(1-x)=-sum^{infty}_{n=1}dfrac{x^n}{n}=-x-dfrac{x^2}{2}-dfrac{x^3}{3}cdots leq -x,;text{for fixed};xin;]0,1[end{align}







        share|cite|improve this answer












        share|cite|improve this answer



        share|cite|improve this answer










        answered Jan 3 at 10:29









        Omojola MichealOmojola Micheal

        1,809324




        1,809324






























            draft saved

            draft discarded




















































            Thanks for contributing an answer to Mathematics Stack Exchange!


            • Please be sure to answer the question. Provide details and share your research!

            But avoid



            • Asking for help, clarification, or responding to other answers.

            • Making statements based on opinion; back them up with references or personal experience.


            Use MathJax to format equations. MathJax reference.


            To learn more, see our tips on writing great answers.




            draft saved


            draft discarded














            StackExchange.ready(
            function () {
            StackExchange.openid.initPostLogin('.new-post-login', 'https%3a%2f%2fmath.stackexchange.com%2fquestions%2f3060375%2fuse-the-concavity-of-log-1-xx-to-show-that-log-1-x-leq-x%23new-answer', 'question_page');
            }
            );

            Post as a guest















            Required, but never shown





















































            Required, but never shown














            Required, but never shown












            Required, but never shown







            Required, but never shown

































            Required, but never shown














            Required, but never shown












            Required, but never shown







            Required, but never shown







            Popular posts from this blog

            Can a sorcerer learn a 5th-level spell early by creating spell slots using the Font of Magic feature?

            Does disintegrating a polymorphed enemy still kill it after the 2018 errata?

            A Topological Invariant for $pi_3(U(n))$